OlimpíadasDivisores

Aqui devem ser postados problemas Olímpicos. Informe a olimpíada e o ano no título do tópico. Exemplo: (OBM - 2008).
Avatar do usuário
majik
sênior
Mensagens: 30
Registrado em: 22 Mar 2014, 15:56
Última visita: 19-07-15
Agradeceu: 2 vezes
Agradeceram: 3 vezes
Set 2014 12 14:23

Divisores

Mensagem não lida por majik »

[tex3]n[/tex3] é um número natural y [tex3]1=d_{1}<d_{2}<....<d_{k}=n[/tex3] seus divisores inteiros positivos, se além [tex3]n=d_{6}^{2}+d_{7}^{2}-1[/tex3] , Halle valores possíveis de [tex3]n[/tex3] .

Editado pela última vez por majik em 12 Set 2014, 14:23, em um total de 2 vezes.
Avatar do usuário
jedi
4 - Sabe Tudo
Mensagens: 985
Registrado em: 11 Jul 2013, 14:57
Última visita: 14-04-24
Agradeceu: 79 vezes
Agradeceram: 745 vezes
Set 2014 12 21:42

Re: Divisores

Mensagem não lida por jedi »

não entendi direito o enunciado mas
uma solução seria

[tex3]n=2\cdot 9\cdot 10=180[/tex3]

[tex3]d_1=2,d_2=3,d_3=4,d_4=5,d_5=6,d_6=9,d_7=10[/tex3]

veja que

[tex3]n=9^2+10^2-1=81+100-1=180[/tex3]

Editado pela última vez por jedi em 12 Set 2014, 21:42, em um total de 2 vezes.
Avatar do usuário
majik
sênior
Mensagens: 30
Registrado em: 22 Mar 2014, 15:56
Última visita: 19-07-15
Agradeceu: 2 vezes
Agradeceram: 3 vezes
Set 2014 13 13:10

Re: Divisores

Mensagem não lida por majik »

Sejam [tex3]1=d_{1}<d_{2}<....<d_{k}=n[/tex3] o conjunto de todos os divisores de um
inteiro positivo [tex3]n[/tex3] . Determine todos os [tex3]n[/tex3] . tais que: [tex3]n=d_{6}^{2}+d_{7}^{2}-1[/tex3] .
Editado pela última vez por majik em 13 Set 2014, 13:10, em um total de 2 vezes.
Avatar do usuário
Auto Excluído (ID:12031)
6 - Doutor
Última visita: 31-12-69
Dez 2014 29 19:07

Re: Divisores

Mensagem não lida por Auto Excluído (ID:12031) »

Seus problemas são os melhores. Vou deixar observações, espero que alguém resolva.

Vamos supor que [tex3]d_6[/tex3] e [tex3]d_7[/tex3] sejam ambos pares, nesse caso [tex3]n[/tex3] é ímpar. Absurdo então [tex3]d_6[/tex3] e [tex3]d_7[/tex3] não são ambos pares.

Na verdade, podemos estender essa idéia pra qualquer natural, não só o número 2. [tex3]d_6[/tex3] e [tex3]d_7[/tex3] são necessariamente primos entre si. Basta supor [tex3]d = mdc(d_6,d_7) \neq 1[/tex3]

[tex3]n\,\,\, mod(d) \equiv -1[/tex3]
logo [tex3]n[/tex3] não seria divisível pelo mdc de dois de seus divisores, o que é absurdo. Então [tex3]d_6[/tex3] e [tex3]d_7[/tex3] são primos entre si.

Também da desigualdade temos que [tex3]1=d_{1}<d_{2}<....<d_{k}=n[/tex3] tiramos que [tex3]d_6 \geq 6[/tex3] e [tex3]d_7 \geq 7[/tex3] também é claro que [tex3]n[/tex3] é diferente de [tex3]d_7[/tex3] se [tex3]d_7=n[/tex3] teríamos que [tex3]n = d_6^2 + n^2 - 1[/tex3] o lado direito é maior que [tex3]n^2[/tex3] , não existe [tex3]n[/tex3] inteiro com mais de [tex3]5[/tex3] divisores que satisfaça essa relação.

[tex3]m\cdot d_6 \cdot d_7 = d_6^2 + d_7^2 -1[/tex3]
[tex3]d_6^2 - m \cdot d_6 \cdot d_7 + d_7^2 -1 =0[/tex3]

o mais intuitivo é tentar resolver essa equação:

[tex3]2d_6 = m\cdot d_7 - \sqrt{(m^2-4)d_7^2 + 4 }[/tex3]
para que [tex3]d_6[/tex3] seja inteiro é necessário que [tex3](m^2-4)d_7^2 = k(k+4)[/tex3] para algum [tex3]k[/tex3] inteiro.
Analogamente [tex3](m^2-4)d_6^2 = k'(k'+4)[/tex3]
De onde:
[tex3]2d_6 = m\cdot d_7 - (k+2)[/tex3]
[tex3]2d_7 = m\cdot d_6 \pm (k'+2)[/tex3]

estou pensando em multiplicar a equação de cima por [tex3]d_6[/tex3] e a de baixo por [tex3]-d_7[/tex3] , sumir com o termo [tex3]m[/tex3] ([tex3]m[/tex3] aqui representa a multiplicação de todos os outros divisores diferentes de [tex3]n[/tex3] ), fazer isso na outra relação: [tex3](m^2-4)d_i^2 = k_i(k_i+4)[/tex3] e ver se dá pra tirar uma relação entre [tex3]d_6[/tex3] e [tex3]d_7[/tex3]

fiz conforme descrevi acima, porém a relação entre [tex3]d_6[/tex3] e [tex3]d_7[/tex3] parece ser complexa:
([tex3]x[/tex3] seria [tex3]d_6[/tex3] , [tex3]y[/tex3] seria [tex3]d_7[/tex3] , [tex3]a[/tex3] seria [tex3]k[/tex3] e [tex3]b[/tex3] seria [tex3]k'[/tex3] )

http://www.wolframalpha.com/input/?i=%7 ... 4%29+solve+
Editado pela última vez por Auto Excluído (ID:12031) em 29 Dez 2014, 19:07, em um total de 2 vezes.
Avatar do usuário
Auto Excluído (ID:12031)
6 - Doutor
Última visita: 31-12-69
Jan 2015 09 09:24

Re: Divisores

Mensagem não lida por Auto Excluído (ID:12031) »

Avancei no problema:
[tex3]n = d_6^2 + d_7^2-1[/tex3]
[tex3]0 \equiv d_6^2 - 1 \,\,\mod(d_7)[/tex3]
[tex3]d_6^2 \equiv 1 \,\, mod(d_7) \rightarrow d_6 \equiv \pm 1\,\,mod(d_7)[/tex3]
[tex3]d_6 = kd_7\pm 1[/tex3] , k é inteiro
do enunciado:
[tex3]0<d_6<d_7[/tex3]
[tex3]0 < kd_7 \pm 1 < d_7[/tex3]
[tex3]\mp 1 < kd_7 < d_7\mp1[/tex3]
como [tex3]d_7 \geq 7 >0[/tex3] veja que se [tex3]k[/tex3] fosse negativo teríamos com certeza [tex3]kd_7 < -1[/tex3] , absurdo!
Devemos ter então [tex3]k\geq 0[/tex3] fácil ver que se [tex3]k\geq 2[/tex3] então [tex3]kd_7 > d_7 \mp 1[/tex3] .
Então ou [tex3]k = 1 \rightarrow d_6 = d_7 - 1[/tex3]
ou [tex3]k=0[/tex3] mas ai: [tex3]d_6 = \pm 1[/tex3] ,absurdo novamente!

Logo: [tex3]\boxed{d_6 = d_7 - 1}[/tex3]
da minha mensagem anterior:
[tex3]2d_6 = m\cdot d_7 - \sqrt{(m^2-4)d_7^2 + 4 }[/tex3]
[tex3]2d_7 - 2 = m\cdot d_7 - \sqrt{(m^2-4)d_7^2 + 4 }[/tex3]
que é verdade se, e somente se: [tex3]m=2[/tex3]
logo, n é da forma: [tex3]n=2\cdot d_6\cdot d_7 = 2\cdot d_6 \cdot (d_6+1)[/tex3]

[tex3]n = d_2 \cdot d_6 \cdot d_7[/tex3]

logo [tex3]d_3,d_4,d_5[/tex3] devem, cada um deles, dividir [tex3]d_6 \cdot d_7[/tex3]
Editado pela última vez por Auto Excluído (ID:12031) em 09 Jan 2015, 09:24, em um total de 2 vezes.
Avatar do usuário
Auto Excluído (ID:12031)
6 - Doutor
Última visita: 31-12-69
Jan 2015 10 12:32

Re: Divisores

Mensagem não lida por Auto Excluído (ID:12031) »

como [tex3]n = 2d_6(d_6+1)[/tex3]
n já tem como divisores: 1,2 e 4.
[tex3]d_2=2[/tex3]
ou:
[tex3]d_3 = 3[/tex3] e [tex3]d_4=4[/tex3]
dessa forma 6 e 12 dividirão n, que terá como divisores: 1,2,3,4,6,12.
o que deixaria [tex3]d_5 = 5[/tex3] ou [tex3]d_5=6[/tex3] se
[tex3]d_5 = 5 \rightarrow d_6=6 \rightarrow d_7 = 7 \rightarrow n= 84[/tex3] que não é divisível por 5.
Então se n for divisível por 3 não será divisível por 5.
[tex3]d_5 = 6 \rightarrow d_6 \in \{7,8,11,12\}[/tex3]
[tex3]d_6 = 7 \rightarrow d_7 = 8 \rightarrow n[/tex3] não é divisível por 3.
[tex3]d_6 = 8 \rightarrow n = 144[/tex3] divisores de [tex3]n:1,2,3,4,6,8,9,12...[/tex3] é solução.
[tex3]d_6 = 11 \rightarrow d_7 = 12[/tex3] porém [tex3]2\cdot11\cdot12[/tex3] é divisível por 8.
então [tex3]n=144[/tex3] foi a única solução tirada daqui.
ou:
[tex3]d_3=4[/tex3]
n não é mais divisível por 3, aliás [tex3]n\equiv d_6 \equiv 1\mod(3)[/tex3]
Se [tex3]d_4,d_5[/tex3] e [tex3]d_6[/tex3] fossem todos primos [tex3]>4[/tex3] teríamos que [tex3]d_7 = d_6+1[/tex3] seria par, e [tex3]\frac{d_7}{2}[/tex3] seria divisor de n. Mas [tex3]\frac{d_7}{2}[/tex3] estaria no intervalo:
[tex3]\{4,...,d_4,d_4+1,...d_5,d_5+1,...,d_6\}[/tex3] ( pois [tex3]d_7>7[/tex3] )
a única forma disso não comprometer a ordem dos divisores é tendo:
[tex3]d_7 = 2d_4[/tex3] ou [tex3]d_7 = 2d_5[/tex3]
[tex3]d_6 = 2d_4-1[/tex3] ou [tex3]d_6 = 2d_5-1[/tex3]
então devemos procurar primos da forma [tex3]p = 2p'-1[/tex3]
Editado pela última vez por Auto Excluído (ID:12031) em 10 Jan 2015, 12:32, em um total de 2 vezes.
Avatar do usuário
Auto Excluído (ID:12031)
6 - Doutor
Última visita: 31-12-69
Jan 2015 11 07:36

Re: Divisores

Mensagem não lida por Auto Excluído (ID:12031) »

sousóeu escreveu: [tex3]d_3=4[/tex3]
n não é mais divisível por 3, aliás [tex3]n\equiv d_6 \equiv 1\mod(3)[/tex3]
Se [tex3]d_4,d_5[/tex3] e [tex3]d_6[/tex3] fossem todos primos [tex3]>4[/tex3] teríamos que [tex3]d_7 = d_6+1[/tex3] seria par, e [tex3]\frac{d_7}{2}[/tex3] seria divisor de n. Mas [tex3]\frac{d_7}{2}[/tex3] estaria no intervalo:
[tex3]\{4,...,d_4,d_4+1,...d_5,d_5+1,...,d_6\}[/tex3] ( pois [tex3]d_7>7[/tex3] )
a única forma disso não comprometer a ordem dos divisores é tendo:
[tex3]d_7 = 2d_4[/tex3] ou [tex3]d_7 = 2d_5[/tex3]
se [tex3]d_7 = 2d_4[/tex3]
como
[tex3]n = 2d_6d_7 = 4d_6d_4[/tex3]
porém [tex3]d_5[/tex3] não pode dividir [tex3]n[/tex3] nesse caso, pois [tex3]d_6[/tex3] e [tex3]d_4[/tex3] são primos.
Analogamente para [tex3]d_7 = 2d_5[/tex3] .
Então não temos [tex3]d_5,d_4,d_6[/tex3] todos primos.
Se só o [tex3]d_6[/tex3] for primo teremos novamente que [tex3]\frac{d_7}{2}[/tex3] divide [tex3]n[/tex3] portanto [tex3]d_7 = 2d_4[/tex3] ou [tex3]d_7 = 2d_5[/tex3]
Editado pela última vez por Auto Excluído (ID:12031) em 11 Jan 2015, 07:36, em um total de 2 vezes.
Avatar do usuário
Auto Excluído (ID:12031)
6 - Doutor
Última visita: 31-12-69
Jan 2015 14 15:44

Re: Divisores

Mensagem não lida por Auto Excluído (ID:12031) »

terminei:
sabendo que [tex3]d_2=2[/tex3] e [tex3]d_3=4[/tex3] :

se [tex3]d_4[/tex3] tiver divisores próprios (tirando o 1), eles só podem ser [tex3]d_2[/tex3] e [tex3]d_3[/tex3] .
Porque do contrário, teríamos um número menor que [tex3]d_4[/tex3] que não é [tex3]d_2[/tex3] nem [tex3]d_3[/tex3] dividindo [tex3]n[/tex3] .
Se [tex3]d_4[/tex3] tiver um divisor maior que [tex3]d_3[/tex3] este divisor será o [tex3]d_4[/tex3] .
Se [tex3]d_4 = 4p = 2^2p[/tex3] , com p primo, [tex3]d_4[/tex3] terá 6 divisores. Absurdo.
Se [tex3]d_4 = p_1p_2[/tex3] então [tex3]p_1[/tex3] divide [tex3]d_4[/tex3] logo é um divisor de [tex3]n[/tex3] menor do que [tex3]d_4[/tex3] logo é 2. Porém [tex3]p_2[/tex3] não existe pois suas únicas opções são [tex3]1[/tex3] ou [tex3]4[/tex3] nenhuma convém. Logo:

[tex3]d_4[/tex3] é primo (maior que 4).

Analogamente:
Os divisores próprios de [tex3]d_5[/tex3] (tirando o 1) só podem ser [tex3]d_2[/tex3] , [tex3]d_3[/tex3] e [tex3]d_4[/tex3] .
Então [tex3]d_5[/tex3] pode ter no máximo 4 divisores próprios, se [tex3]d_5 = 4p[/tex3] teríamos novamente 5 divisores próprios para [tex3]d_5[/tex3] , absurdo.
Ou [tex3]d_5[/tex3] é primo, ou então:
[tex3]d_5 = 2d_4[/tex3] ou [tex3]d_5 = d_4^2[/tex3] .

Para [tex3]d_6[/tex3] :
Os divisores próprios ([tex3]\neq 1[/tex3] ) de [tex3]d_6[/tex3] podem ser [tex3]d_2[/tex3] , [tex3]d_3[/tex3] , [tex3]d_4[/tex3] e [tex3]d_5[/tex3] . [tex3]d_6[/tex3] tem no máximo 6 divisores positivos.

[tex3]d_6[/tex3] poderia ter as formas:

[tex3]d_6 = p_1p_2p_3[/tex3] , porém nesse caso [tex3]d_6 = 2p_2p_3[/tex3] se [tex3]p_2[/tex3] não for nem [tex3]d_4[/tex3] nem [tex3]d_5[/tex3] ele vai ser um novo divisor de [tex3]n[/tex3] , nesse caso [tex3]d_4[/tex3] e [tex3]d_5[/tex3] teriam que ser primos e [tex3]d_6 = 2d_4d_5[/tex3] porém [tex3]2d_4[/tex3] divide [tex3]d_6[/tex3] e [tex3]d_5 \neq 2d_4 > d_4[/tex3] ([tex3]2d_4[/tex3] seria um divisor de [tex3]n[/tex3] maior que [tex3]d_4[/tex3] e menor que [tex3]d_6[/tex3] diferente de [tex3]d_5[/tex3] ), absurdo. Então [tex3]d_6[/tex3] não tem essa forma.

[tex3]d_6 = p_1^2p_2[/tex3] ,
Se [tex3]p_1 = d_4[/tex3] e [tex3]p_2=2[/tex3] teríamos que tanto [tex3]2d_4[/tex3] como [tex3]d_4^2[/tex3] dividem [tex3]d_6[/tex3] ao mesmo tempo, ambos são maiores que [tex3]d_4[/tex3] e menores que [tex3]d_6[/tex3] então ambos deveriam ser [tex3]d_5[/tex3] absurdo.
Se [tex3]p_1=d_4[/tex3] e [tex3]p_2=d_5[/tex3] , [tex3]d_4d_5[/tex3] dividiria [tex3]d_6[/tex3] absurdo.
[tex3]p_1[/tex3] não pode ser [tex3]d_4[/tex3] .
analogamente [tex3]p_1[/tex3] não pode ser [tex3]d_5[/tex3] .
Neste caso [tex3]p_1=2[/tex3] :
[tex3]d_6 = 4p_2[/tex3] , se [tex3]p_2[/tex3] fosse [tex3]d_4[/tex3] , [tex3]d_5 = 2d_4[/tex3] . De fato esse é o único modo. [tex3]d_6= 4d_4[/tex3] , com [tex3]d_4[/tex3] primo e [tex3]d_5=2d_4[/tex3] . Só que neste caso:
[tex3]n = 8d_4(4d_4+1)[/tex3]
infelizmente [tex3]8[/tex3] divide n o que significa que [tex3]d_4 = \{5,7\}[/tex3] porém neste caso, [tex3]d_5=8\neq 2d_4[/tex3] . Absurdo. Logo [tex3]d_6[/tex3] não tem essa forma.

[tex3]d_6 = p_1p_2[/tex3] Neste caso, mais geral:
[tex3]\begin{cases}
d_6=2d_4 \\
d_6=2d_5 \,\,\,\,\,(\,d_5\,\,primo)\\
d_6=d_4d_5\,\,\,\, (d_5 primo)
\end{cases}[/tex3]
primeira hipótese:
[tex3]d_6 = 2d_4 \rightarrow n = 4d_4(2d_4+1)[/tex3] de onde [tex3]d_5[/tex3] ou é um primo que divide [tex3]2d_4+1[/tex3] para isso:
[tex3]2d_4+1 = p_1p_2...d_5...p_k > p_1p_2...d_4...p_k \rightarrow p_1p_2...p_k <2 + \frac{1}{d_4}[/tex3]
[tex3]p_1p_2...p_k = \{1,2\}[/tex3]
porém se o produto for [tex3]1[/tex3] teremos [tex3]d_7=d_5[/tex3] e se o produto for 2 teremos um par igual a um ímpar.
Ou [tex3]d_5=2d_4 = d_6[/tex3] absurdo também.
ou [tex3]d_5=d_4^2 \rightarrow \frac{n}{d_5} = (8 + \frac{4}{d_4})[/tex3] , porém [tex3]d_4[/tex3] não divide 4.
Logo [tex3]d_6 \neq 2d_4[/tex3] .
Se [tex3]d_6 = 2d_5[/tex3] [tex3]\rightarrow n =4d_5(2d_5+1)[/tex3] de onde [tex3]d_4[/tex3] deve dividir o número ímpar [tex3]2d_5+1[/tex3] , porém neste caso como [tex3]d_4>3 \rightarrow \frac{2d_5+1}{d_4} < d_5[/tex3] então temos um divisor de [tex3]n[/tex3] menor do que [tex3]d_5[/tex3] logo:
[tex3]\begin{cases}
2d_5+1=2d_4 \,\,(absurdo)\\
2d_5 + 1 =4d_4 \,\,(absurdo\,\,pela\,\,paridade)\\
2d_5+1=d_4^2 \,\,(absurdo:
\end{cases}[/tex3]
se [tex3]2d_5+1 = d_4^2[/tex3] e lembrando que [tex3]n[/tex3] não é múltiplo de 3, logo [tex3]d_4[/tex3] e [tex3]d_5[/tex3] também não o são: teremos aplicando módulo 3 dos dois lados da igualdade:
[tex3]2d_5 +1 \equiv 1 \mod(3)[/tex3] absurdo! Logo [tex3]d_6 \neq 2d_5[/tex3]
finalmente:
[tex3]d_6=d_4d_5[/tex3] com ambos primos. Neste caso [tex3]n = 2d_4d_5(d_4d_5+1)[/tex3] porém: [tex3]d_5<2d_5<d_6[/tex3] e [tex3]2d_5[/tex3] divide [tex3]n[/tex3] absurdo. Logo [tex3]d_6[/tex3] não tem essa forma.

[tex3]d_6[/tex3] é primo. (Neste caso já mostrei que não temos [tex3]d_4[/tex3] e [tex3]d_5[/tex3] primos ao mesmo tempo)
restam analisar [tex3]d_5 = 2d_4[/tex3] e [tex3]d_5=d_4^2[/tex3]
se [tex3]d_5 = d_4^2[/tex3] teríamos o problema de [tex3]d_4<2d_4<d_5[/tex3] dividir [tex3]n[/tex3] .
Resta que [tex3]d_5 = 2d_4[/tex3]
bom, se [tex3]d_6[/tex3] é primo e [tex3]d_4[/tex3] divide [tex3]n[/tex3] então [tex3]d_4[/tex3] divide [tex3]2d_6(d_6+1)[/tex3] logo [tex3]\boxed{d_6+1 = 2d_4x}[/tex3] com [tex3]x\in \mathbb{N}[/tex3] note que [tex3]x<d_6[/tex3]
teríamos então [tex3]n = 4d_6d_4x = 4(2d_4x-1)d_4x[/tex3]
[tex3]x[/tex3] é divisor de [tex3]n[/tex3] menor do que [tex3]d_6[/tex3] . [tex3]x\in \{1,2,4,d_4,2d_4\}[/tex3]
Se [tex3]x[/tex3] for par [tex3]8[/tex3] dividirá [tex3]n[/tex3] logo [tex3]d_4 \in \{5,7\}[/tex3] porém [tex3]d_5=8[/tex3] absurdo.
Logo [tex3]x[/tex3] é ímpar, ou [tex3]x = 1[/tex3] ou [tex3]x=d_4[/tex3]
Se [tex3]x=d_4[/tex3] [tex3]d_6 = 2d_4^2-1 >d_4^2 >d_5[/tex3] porém [tex3]d_4^2[/tex3] divide [tex3]n[/tex3] absurdo.
[tex3]x=1 \rightarrow d_6<d_5[/tex3] absurdo.
Logo [tex3]d_6[/tex3] não é primo.

Está provado que n=144 é solução única.

Editado pela última vez por Auto Excluído (ID:12031) em 14 Jan 2015, 15:44, em um total de 2 vezes.
Responder
  • Tópicos Semelhantes
    Resp.
    Exibições
    Últ. msg
  • Nova mensagem Divisores
    por nathyjbdl » » em Ensino Fundamental
    3 Resp.
    674 Exibições
    Últ. msg por mateusITA
  • Nova mensagem Divisores
    por AnnaBeatriz » » em Ensino Médio
    1 Resp.
    633 Exibições
    Últ. msg por Auto Excluído (ID:12031)
  • Nova mensagem (Rumo ao ITA) - Divisores
    por emanuel9393 » » em IME / ITA
    1 Resp.
    3626 Exibições
    Últ. msg por Auto Excluído (ID:12031)
  • Nova mensagem Total de divisores de um número
    por ludwing » » em Ensino Médio
    4 Resp.
    3529 Exibições
    Últ. msg por csmarcelo
  • Nova mensagem Divisão em N, Múltiplo e divisores em Z, Número Primo e Comp
    por marmarcela » » em Ensino Médio
    2 Resp.
    1944 Exibições
    Últ. msg por marmarcela

Voltar para “Olimpíadas”